ChaseDream

标题: 【求助GWD TN24 16 Q3~谢谢!~】 [打印本页]

作者: 水色青空    时间: 2010-3-5 12:57
标题: 【求助GWD TN24 16 Q3~谢谢!~】
3. GWD25-Q3.
Mel: The official salary for judges has always been too low to attract the best candidates to the job. The legislature’s move to raise the salary has done nothing to improve the situation, because it was coupled with a ban on receiving money for lectures and teaching engagements.
Pat:  No, the raise in salary really does improve the situation. Since very few judges teach or give lectures, the ban will have little or no negative effect.
Pat’s response to Mel is inadequate in that it
  1. attempts to assess how a certain change will affect potential members of a group by providing evidence about its effect on the current members.
  2. mistakenly takes the cause of a certain change to be an effect of that change
  3. attempts to argue that a certain change will have a positive effect merely by pointing to the absence of negative effects
  4. simply denies Mel’s claim without putting forward any evidence in support of that denial
  5. assumes that changes that benefit the most able members of a group necessarily benefit all members of that group.










    答案选A~~我选的是C~~~不明白,希望好心人解释~谢谢了~~


作者: 水色青空    时间: 2010-3-5 19:21
顶一下~
作者: 水色青空    时间: 2010-3-6 19:14
顶~~~~
作者: hutian1992    时间: 2012-8-17 14:25
c选项看似正确,其实不然
“attempts to argue that a certain change will have a positive effect merely by pointing to the absence of negative effects”
但是pat是认为因为the ban没有negative effects,所以 the raise有positive effects
c项的错误是将两个the ban 和the raise揉到一起了




欢迎光临 ChaseDream (https://forum.chasedream.com/) Powered by Discuz! X3.3